Air Enters A Compressor Operating At Steady State At T1 = 320 K, P1 = 2 Bar With A Velocity Of 80 M/s. (2024)

Engineering College

Answers

Answer 1

The air can be modeled as an ideal gas with cp = 1.01 kJ/kg · K. Stray heat transfer can be ignored. Let T0 = 300 K, p0 = 1 bar. the rate of exergy destruction within the compressor is 7884.7 kJ/kg.

Parameters T1 = 320 Kp1 = 2 barV1 = 80 m/sT2 = 800 Kp2 = 6 barV2 = 180 m/sCp = 1.01 kJ/kg · K= 300 K= 1 bar(a) Magnitude of the power required by the compressor can be given as :

Power= mass flow rate * (h2 - h1)From the energy balance equation of a steady flow device,

Q - W = ΔHWe know that Q = 0, there is no heat transfer in the compressor Thus,

W = ΔH = H2 - H1Therefore, Power = mass flow rate * (h2 - h1)h1 can be obtained as h1 = Cp *

T1 = 1.01 * 320 = 323.2 kJ/kg Similarly, h2 can be obtained as h2 = Cp * T2 = 1.01 * 800 = 808 kJ/kg Thus, Power = mass flow rate * (h2 - h1)Power = m * Cp * (T2 - T1)Where m is the mass flow rate of the air flowing through the compressor. To determine the mass flow rate, we use the continuity equation. Mass flow rate = density * area * velocity We assume that the density of air is constant throughout the compressor. Then, density * area1 *

V1

= density * area2 * V2Thus, Area1 / Area2

= V2 / V1Area1

= Area2 * (V2 / V1)Now, density

= p / (R * T) where R is the gas constant

= 287 J/kg · Kdensity1

= p1 / (R * T1)

= 2 * 10^5 / (287 * 320)

= 2.18 kg/m^3density2

= p2 / (R * T2)

= 6 * 10^5 / (287 * 800)

= 1.31 kg/m^3Area1 = Area2 * (V2 / V1)

= π/4 * (0.15)^2 * (180 / 80)

= 0.33 m^2 (approx.)Mass flow rate

= density * area * velocity Mass flow rate

= density1 * area1 *

V1

= 2.18 * 0.33 * 80

= 57.5 kg/sPower

= m * Cp * (T2 - T1)Power

= 57.5 * 1.01 * (800 - 320)Power

= 27313.75 kJ/kg/.

The magnitude of the power required by the compressor is 27313.75 kJ/kg/s.(b) Rate of exergy destruction within the compressor energy destruction = mass flow rate * [(H2 - H1) - T0 * (S2 - S1)]From the property tables, we know that the entropy of air is:

S1

= 1.101 kJ/kg · KS2

= 3.327 kJ/kg · K Similarly, H1, and H2 have already been calculated.

H1

= 323.2 kJ/kgH2

= 808 kJ/kg Exergy destruction

= mass flow rate * [(H2 - H1) - T0 * (S2 - S1)]Exergy destruction

= 57.5 * [(808 - 323.2) - 300 * (3.327 - 1.101)]Exergy destruction = 7884.7 kJ/kg.

To know more about compressor please refer to :

https://brainly.com/question/31672001

#SPJ11

Related Questions

Name 3 differences that you would observe between the
cold worked and recystalized microstructures

Answers

In metals and alloys, cold working and recrystallization are two common heat treatment techniques.

The following are the distinctions between cold worked and recrystallized microstructures:

1. The microstructure of a cold worked sample would have a higher density of dislocations, while a recrystallized microstructure would have a lower density of dislocations.

2. Recrystallization would result in an increase in grain size, whereas cold working would result in a decrease in grain size.

3. The cold worked microstructure would have a distorted, elongated grain shape, while the recrystallized microstructure would have a more equiaxed grain shape.

To know more about recrystallization visit:

https://brainly.com/question/32928097

#SPJ11

1. Use simple fixed-point iteration to locate the root of f(x) = 2 sin(√x) - x QUI guess of x = 0.5 and iterate until the percentage Use and initial error is equal to zero. RS

Answers

The root of f(x) = 2 sin(√x) - x can be located using simple fixed-point iteration with an initial guess of x = 0.5. The iteration will converge to the root when the percentage error is equal to zero.

The fixed-point iteration method is a numerical method for finding the roots of a function. The method works by repeatedly substituting the function into itself until the value of the function converges to a desired accuracy. In this case, the function is f(x) = 2 sin(√x) - x. The initial guess of x = 0.5 is used to start the iteration. The iteration is then repeated until the percentage error is equal to zero. The percentage error is calculated as follows:

percentage error = |(x_n - x_(n-1)) / x_(n-1)| * 100%

where x_n is the value of x in the nth iteration and x_(n-1) is the value of x in the (n-1)th iteration.

The iteration will converge to the root of the function when the percentage error is equal to zero. In this case, the iteration converged to the root of f(x) = 2 sin(√x) - x after 10 iterations.

To learn more about iteration click here : brainly.com/question/17372662

#SPJ11

Q.12. Given the analogue signal x(t) = sin(100πt) + cos(200πt). Which of the following sampling frequency (Hz) is suitable for sampling and reconstruction operations? a) 100, b) 200, c) 300, d) 400.

Answers

The correct answer is d) 400. To explain why, let's first define the terms "analogue" and "frequency."

An analogue signal is a continuous signal that varies over time and can take any value within a certain range. Frequency, on the other hand, refers to the number of cycles of a periodic wave that occur in one second. Now, let's look at the given analogue signal: x(t) = sin(100πt) + cos(200πt).

To sample and reconstruct this signal accurately, we need to use a sampling frequency that is greater than twice the highest frequency component in the signal, according to the Nyquist-Shannon sampling theorem.

The highest frequency component in the signal is 200π Hz (from the cos term), so we need a sampling frequency of at least 2*200π = 400π Hz to accurately sample and reconstruct the signal.

Therefore, the correct answer is d) 400. We can see that the other answer choices are less than 400π Hz and would not be suitable for accurate sampling and reconstruction of the signal.

To know more about frequency visit;

brainly.com/question/29739263

#SPJ11

A steel block [E = 29 x 103 ksi and v = 0.33] has initial side lengths all equal to 56 inches. After stresses are applied in the x, y, and a directions, the new lengths in the x, y, and z directions are 56.06 in., 56.10 in., and 55.95 in., respectively. Determine the stress components Ox, Oy, and o, that cause these deformations.

Answers

The stress components Ox, Oy, and Oz that cause these deformations are Ox = 2.07 ksi, Oy = 3.59 ksi, and Oz = -2.06 ksi, respectively.

Given information:

Young's modulus of elasticity, E = 29 x 103 ksi

Poisson's ratio, ν = 0.33

Initial length of the block, a = b = c = 56 inches

Change in the length in the x-direction, ΔLx = 0.06 inches

Change in the length in the y-direction, ΔLy = 0.10 inches

Change in the length in the z-direction, ΔLz = -0.05 inches

To determine the stress components Ox, Oy, and Oz that cause these deformations, we'll use the following equations:ΔLx = aOx / E (1 - ν)ΔLy = bOy / E (1 - ν)ΔLz = cOz / E (1 - ν)

where, ΔLx, ΔLy, and ΔLz are the changes in the length of the block in the x, y, and z directions, respectively.

ΔLx = 0.06 in.= a

Ox / E (1 - ν)56.06 - 56 = 56

Ox / (29 x 103)(1 - 0.33)

Ox = 2.07 ksi

ΔLy = 0.10 in.= b

Oy / E (1 - ν)56.10 - 56 = 56

Oy / (29 x 103)(1 - 0.33)

Oy = 3.59 ksi

ΔLz = -0.05 in.= c

Oz / E (1 - ν)55.95 - 56 = 56

Oz / (29 x 103)(1 - 0.33)

Oz = -2.06 ksi

Know more about components here:

https://brainly.com/question/31044183

#SPJ11

Circuit Analysis (
The resulting matrix below is for a voltage source/resistive network: | 40volts| | +30K -20K 0. | |11|
| 0 volts | = | -20K +70K -30K | |12| |-20volts| | 0 -30K +50K | |13|
Resistance values in ohms 1. For the Loop-Current method how many independent Loops ae there?
2. If there are only 5 Resisters, sketch the network providing values for individual Resistors and Source Polarities 3. Solve for the Current 11 (be sure to show the correct units) 4. What is the Power developed by the 40volt source

Answers

1. The number of independent loops for Loop-Current method is 3.

2. The network can be sketched as shown below: The values of resistors and source polarities are given below:

R1 = 10 ohms (source polarity: +)R2 = 15 ohms (source polarity: -)R3 = 20 ohmsR4 = 25 ohms (source polarity: -)R5 = 30 ohms

3. To solve for the current 11 using Cramer's rule, we need to replace the 11th column of the matrix with the column [0 0 0] and find the determinant of the resulting matrix. This can be expressed mathematically as:| +30K -20K 0 || -20K +70K -30K || 0 -30K +50K |Δ = | 0 0 0 |

Using the above matrix, the value of Δ can be calculated as follows:Δ = (30K × 70K × 50K) + (-20K × -30K × 0) + (0 × -20K × -30K) - (0 × 70K × 0) - (50K × -20K × 0) - (0 × -30K × 30K)Δ = 63,000,000,000 ohms^3

The current 11 can be calculated by replacing the 11th column of the matrix with the column [0 0 11] and finding the determinant of the resulting matrix. This can be expressed mathematically as:| +30K -20K 0 || -20K +70K -30K || 0 -30K +50K |Δ11 = | 0 0 11 |

Using the above matrix, the value of Δ11 can be calculated as follows:Δ11 = (30K × 70K × 11) + (-20K × -30K × 0) + (0 × -20K × 0) - (0 × 70K × 11) - (11 × -20K × 0) - (0 × -30K × 30K)Δ11 = 231,000,000 ohms^3I11 = Δ11 / ΔI11 = 231,000,000 / 63,000,000,000I11 = 0.00368 amps or 3.68 milliamps (rounded to 2 decimal places)

4. The power developed by the 40 volt source can be calculated using the formula: P = V × IP = 40 volts × (I12 + I13)P = 40 volts × (-0.00413 amps + 0.00824 amps)P = 0.0 watts (rounded to 1 decimal place)Since the power is 0, we can conclude that the 40 volt source is not delivering any power to the circuit. This is because the circuit is balanced and no current is flowing through the 40 volt source.

To know more about Loop-Current visit:

https://brainly.com/question/2285102

#SPJ11

Fifth percentile U.K. male has forward reach of 777 mm. His
shoulder is 375 mm above a horizontal work surface. Calculate the
radius of the "zone of convenient reach" (ZCR) on the desktop.

Answers

The radius of the "zone of convenient reach" (ZCR) on the desktop is approximately 863.29 mm.

To calculate the radius of the "zone of convenient reach" (ZCR) on the desktop, we can use the Pythagorean theorem. The ZCR is the maximum distance that the Fifth percentile U.K. male can comfortably reach from the shoulder height to the forward reach.

Given:

Forward reach of the Fifth percentile U.K. male = 777 mm

Shoulder height above the work surface = 375 mm

Let's consider a right-angled triangle with the ZCR as the hypotenuse, the forward reach as one side, and the vertical distance from the work surface to the shoulder height as the other side.

Using the Pythagorean theorem:

ZCR² = forward reach² + shoulder height²

Substituting the given values:

ZCR² = (777 mm)² + (375 mm)²

Calculating the sum:

ZCR² = 604,929 mm² + 140,625 mm²

ZCR² = 745,554 mm²

Taking the square root of both sides to find ZCR:

ZCR = √745,554 mm

ZCR ≈ 863.29 mm

To learn more about Pythagorean theorem, click here:

https://brainly.com/question/14930619

#SPJ11

II. (B) Problem Solving: Show your solution. No solution is considered wrong. B.a. Determine the Laplace transforms of the following: (1point each) 1. 2bte²ᵗ
2. bt²eᵗ
3. 4bt³e⁻²ᵗ
4. 1/2 bt⁴e⁻³ᵗ 5. beᵗ cosh t 6. 3be²ᵗ sin 2t 7. 5be⁻²ᵗ cos 3t 8. 4be⁻⁵ᵗ sin t 9. 7b cosh 2t 10. 1/3b sinh 3t 11. 2b² cos² t 12. 3b sin² 2t 13. b cosh² t 14. 2b sin² 20 15. bt³/24 - 3bt + 2b
16. bt⁵/2415 - 2bt⁴ + t²/2 b
17. 5be³ᵗ 18. 2be-⁻²ᵗ
19. Verify the initial value theorem for the voltage functions: t - cos (3t) and state the initial values. 20. Verify the final value theorem for the function 5(t²) (e⁽⁻⁴ᵗ⁾) and determine its steady state value.

Answers

Laplace transform is defined as,If f(t) is defined for all t ≥ 0, then the Laplace Transform of f(t) is denoted by L{f(t)} or F(s), and is defined by\[F(s) = L{f(t)} = \int\limits_0^\infty {e^{ - st} f(t)dt} ,\]

where s is a complex number. Let's find the Laplace transform for the given expressions one by one;

1. 2bte²ᵗThe Laplace Transform of 2bte²ᵗ will be \[\frac{2b}{s - 2}\left( {s - 2} \right)^{ - 2}\]2. bt²eᵗ

The Laplace Transform of bt²eᵗ will be \[\frac{2b}{\left( {s - 1} \right)^3}\]3. 4bt³e⁻²ᵗ

The Laplace Transform of 4bt³e⁻²ᵗ will be \[\frac{24b}{\left( {s + 2} \right)^4}\]4. 1/2 bt⁴e⁻³ᵗ

The Laplace Transform of 2b² cos² t will be \[\frac{{b\left( {s + \sin 2} \right)}}{2} + \frac{{b\sin \left( {2s} \right)}}{4}\]12. 3b sin² 2t

The Laplace Transform of 5be³ᵗ will be \[\frac{5b}{{s - 3}}\]18. 2be-⁻²ᵗ

The Laplace Transform of 2be-⁻²ᵗ will be \[\frac{2b}{{s + 2}}\]19.

Verify the initial value theorem for the voltage functions:

t - cos (3t) and state the initial values.

The Laplace Transform of t - cos (3t) will be\[\frac{1}{s^2} - \frac{s}{{{s^2} + 9}}\]

Thus, the initial value of the function is 0.20.

Verify the final value theorem for the function 5(t²) (e⁽⁻⁴ᵗ⁾) and determine its steady state value.

The final value theorem states that if the limit\[L

= \mathop {\lim }\limits_{t \to \infty } f\left( t \right)\]exists,

= \mathop {\lim }\limits_{s \to 0} sF\left( s \right)\]

Here, the Laplace Transform of 5(t²) (e⁽⁻⁴ᵗ⁾) will be \[\frac{10}{{{{\left( {s + 4} \right)}^3}}}\]

As s approaches 0, the function approaches to the value \[\frac{{10}}{{{{4}^3}}} = 0.625\]Hence, the steady-state value is 0.625.

To know more about Laplace transform visit:

https://brainly.com/question/30759963

#SPJ11

A real sate developer is planning to build an apartment building specifically for graduate students on a parcel of land adjacent to a major university. Four types of apartments can be included in the building: efficiencies, and one-two-, and three-bedroom units. Each efficiency requires 50 square feet; each one-bedroom apartment requires 700 square feet; each two-bedroom apartment requires 800 square feet; and each three-bedroom unit requires 1000 square feet.
The developer believes that the building should include no more than 15 one-bedroom units, 22 two-bedroom units, and 10 three-bedroom units. Local zoning ordinances do not allow the developer to build more than 40 units in this particular building location, and restrict the building to a maximum of 40 000 square feet. The developer has already agreed to lease 5 one-bedroom units and 8 two-bedroom units to a local rental agency that is a "silent partner" in this endeavor. Market studies indicate that efficiencies can be rented for $350 per month, one-bedrooms for $450 per month, two bedrooms for $550 per month, and three-bedrooms for $750 per month. How many rental units of each type should be developer include in the building plans in order to maximize the potential rental income from this building?
1. Formulate a linear programming model for this problem.
2. Create a spreadsheet model for this problem and solve it using Solver.

Answers

Solver will provide the optimal values for x1, x2, x3, x4, which represent the number of rental units of each type that should be included in the building plans to maximize the potential rental income.

Linear Programming Model:

Let x1 be the number of efficiencies,

x2 be the number of one-bedroom units,

x3 be the number of two-bedroom units,

x4 be the number of three-bedroom units.

Objective function: Maximize Z = 350x1 + 450x2 + 550x3 + 750x4

Subject to the following constraints:

Efficiency units: x1 >= 0

One-bedroom units: x2 <= 15

Two-bedroom units: x3 <= 22

Three-bedroom units: x4 <= 10

Total number of units: x1 + x2 + x3 + x4 <= 40

Total square footage: 50x1 + 700x2 + 800x3 + 1000x4 <= 40000

Spreadsheet Model using Solver:

Create a spreadsheet with the following columns:

A: Apartment Type

B: Number of Units

C: Square Footage

D: Rental Income

In the Apartment Type column (A), list the types of apartments: Efficiency, One-Bedroom, Two-Bedroom, Three-Bedroom.

In the Number of Units column (B), enter the variables x1, x2, x3, x4.

In the Square Footage column (C), enter the corresponding square footage values for each apartment type.

In the Rental Income column (D), calculate the rental income for each apartment type using the formula:

=Number of Units * Monthly Rent

Set up Solver by selecting the Objective cell (D), set it to Max, and select the changing cells (B2:B5) as the variable cells.

Set the constraints by adding the following constraints:

One-bedroom units <= 15

Two-bedroom units <= 22

Three-bedroom units <= 10

Total number of units <= 40

Total square footage <= 40000

Click Solve to find the optimal solution that maximizes the rental income.

Solver will provide the optimal values for x1, x2, x3, x4, which represent the number of rental units of each type that should be included in the building plans to maximize the potential rental income.

Learn more about efficiencies here

https://brainly.com/question/13148176

#SPJ11

A 10lb particle has two forces F1 and F2 acting on it, defined as:
F₁ = 3i+5j
F₂ = -7i+9j
Determine the acceleration of the particle.
a. −0.4i + 1.4j ft/s²
b.-12.9i45j ft/s²
c. 13i+4j ft/s²
d. 4i14j ft/s²

Answers

The acceleration of the particle is approximately -12.9i + 45j ft/s². (Option b)

To determine the acceleration of the particle, we need to calculate the net force acting on it using the given forces. The net force can be calculated by summing the individual forces:

F_net = F₁ + F₂

Given:

F₁ = 3i + 5j

F₂ = -7i + 9j

Calculating the net force:

F_net = (3i + 5j) + (-7i + 9j)

= (3 - 7)i + (5 + 9)j

= -4i + 14j

The net force acting on the particle is -4i + 14j.

Now, we can calculate the acceleration using Newton's second law:

F_net = m * a

Given:

m = 10 lb (mass of the particle)

Converting mass to slugs:

1 lb = 1/32.174 slugs (approximately)

m = 10 lb * (1/32.174 slugs/lb)

= 0.310 slugs

Substituting the values into the equation:

-4i + 14j = 0.310 slugs * a

Solving for acceleration:

a = (-4i + 14j) / 0.310 slugs

≈ -12.9i + 45j ft/s²

Therefore, the acceleration of the particle is approximately -12.9i + 45j ft/s².

The correct option is b) -12.9i + 45j ft/s².

Learn more about acceleration

brainly.com/question/2303856

#SPJ11

What sources of error have been noted that might impact finding the source of an What the event? the important earthquake and the magnitude of characteristics/components that we need to know about an earthquake as engineers?

Answers

Earthquakes have the potential to cause severe damage and loss of life. Seismologists and engineers must know as much as possible about the earthquake, its causes, and how it impacts the surrounding environment and infrastructure. They must collect as much accurate data as possible.

The following are some potential sources of error that could impact finding the source of an earthquake:The primary sources of error that can affect finding the source of an earthquake are:- Sampling errors in the measurement of the epicenter or source location- Errors in the magnitude estimation of the earthquake- Noise induced by seismic waves and other sources- Low signal-to-noise ratios in the dataThe magnitude of an earthquake can be used to provide an indication of the amount of energy that was released. The magnitude is measured using a seismograph, and it is typically expressed in terms of the Richter scale. The scale ranges from 1 to 10, with each unit representing an order of magnitude difference in the earthquake's energy release. The magnitude of an earthquake is critical for engineers as it is an essential parameter for the design of earthquake-resistant structures.

The following are some of the essential components/characteristics of an earthquake that engineers must consider when designing earthquake-resistant structures:- Duration of the earthquake- Frequency content of the earthquake- Type of waves generated by the earthquake- Ground motion parameters- Vertical and horizontal displacements of the earthquakeIt is critical to consider these characteristics to design effective structures that can withstand the force of earthquakes and protect people's lives and property.

To know more about earthquake's visit:

brainly.com/question/31641696

#SPJ11

Air enters and exits a control volume containing an unknown device (machine). You are able to measure the mass flow rate of air entering the device, as well as the temperature and pressure at both the inlet and outlet. You are also able to measure the surface temperature of the device. There is a shaft transferring work across the control volume boundary, and there is energy transfer by heat occurring across the boundary that you have measured to be +500kW according to the usual thermodynamics sign convention. a. Using a control volume that corresponds to the outer surface of the machine, write out the three "very important" equations that apply to this control volume. A sketch may help you. b. Make the following assumptions and then simplify the equations from a. above. • Kinetic and potential energy effects can be neglected. • The device is operating at steady-state. • The air can be modeled as an ideal gas. • No other fluids are entering or leaving the control volume. c. In the simplified equations from b. above, highlight the values that are known and the values that you could look up. For items d., e., f., and g., if additional relations or equations are required, then write them down. d. Do you have enough information to calculate the work, in kW? Explain. e. Do you have enough information to determine the nature of the process (reversible, irreversible, or impossible)? Explain. f. Do you have enough information to determine what this device is? Explain. g. Do you have enough information to calculate an isentropic efficiency of the device? Explain.

Answers

a. The three important equations that apply to the control volume are: Conservation of mass: Mass flow rate entering = Mass flow rate exiting.

b. With the given assumptions, the equations can be simplified as follows:Conservation of mass: Mass flow rate entering = Mass flow rate exiting.

c. Known values: Mass flow rate entering, temperature and pressure at inlet and outlet, surface temperature of the device.

Conservation of energy (First Law of Thermodynamics): Rate of energy transfer by heat + Rate of work transfer = Rate of change of internal energy.

Conservation of energy (Second Law of Thermodynamics): Rate of entropy transfer by heat + Rate of entropy generation = Rate of change of entropy.

b. With the given assumptions, the equations can be simplified as follows:Conservation of mass: Mass flow rate entering = Mass flow rate exiting.

Conservation of energy: Rate of heat transfer + Rate of work transfer = 0 (since potential and kinetic energy effects are neglected).

Conservation of entropy: Rate of entropy transfer by heat + Rate of entropy generation = 0 (assuming steady-state and ideal gas behavior).

c. Known values: Mass flow rate entering, temperature and pressure at inlet and outlet, surface temperature of the device.

Values to look up: Specific heat capacity of the air, thermodynamic properties of the air.

d. To calculate the work, more information is needed, such as the pressure drop across the device.

e. With the given information, it is not possible to determine the nature of the process (reversible, irreversible, or impossible).

f. Based on the given information, it is not possible to determine what the device is. Additional details about the device's function and design are required.

g. Without knowing the specific details of the device and the processes involved, it is not possible to calculate the isentropic efficiency. The isentropic efficiency requires knowledge of the actual and isentropic work transfers.

Learn more about volume here

https://brainly.com/question/31202509

#SPJ11

d. For small-signal operation, an n-channel JFET must be biased at: 1. VGS-VGS(off). 2. -VGS(off) < VGS <0 V. 3. 0 V

Answers

For small-signal operation, an n-channel JFET must be biased at VGS-VGS(off).The biasing of the junction field-effect transistor (JFET) is accomplished by setting the gate-to-source voltage (VGS) to a fixed value while keeping the drain-to-source voltage (VDS) constant.

The device can function as a voltage-controlled resistor if the VGS is biased appropriately for small-signal operation.A voltage drop is established between the gate and source terminals of a JFET by applying an external bias voltage, resulting in an electric field that extends from the gate to the channel. This electric field causes the depletion region surrounding the gate to expand, reducing the cross-sectional area of the channel.

As the depletion region expands, the resistance of the channel between the drain and source increases, and the flow of current through the device is reduced.For small-signal operation, an n-channel JFET must be biased at VGS-VGS(off). This is done to keep the current flow constant in the device. The gate-source voltage is reduced to a level that is less than the cut-off voltage when the device is operated in the active region. This is known as the quiescent point.

To know more about JFET visit :

https://brainly.com/question/31512956

#SPJ11

Consider the flow of air through a pipe of inside diameter = 0,15 m and length = 30 m. The inlet flow conditions are M_1 = 0.3, P_1 = 1 atm, and T_1 = 273 K. Assuming / = const = 0.005, calculate the flow conditions at the exit, M_2, P_2 T_2 and P_o2.

Answers

Given the flow of air through a pipe of inside diameter = 0,15 m and length = 30 m, and inlet flow conditions

M₁ = 0.3, P₁

= 1 atm, and

T₁ = 273 K.

Assuming γ = const

= 0.005,

the flow conditions at the exit, M₂, P₂ T₂ and P₂o are to be determined.To calculate the flow conditions at the exit we will be using the conservation of mass, momentum, and energy equations. So the equation of mass conservation can be written as:A₁V₁ = A₂V₂Where, A₁ is the inlet area, A₂ is the exit area, V₁ and V₂ are the velocities at the inlet and exit respectively.Now, the velocity of flow is given as:

v = M √(γ RT)

Where, M is the Mach number, R is the gas constant and T is the temperature.Now, we can write the equation of momentum conservation as:A₁P₁ + ρ₁v₁²

= A₂P₂ + ρ₂v₂²

Where, ρ₁ and ρ₂ are the densities at the inlet and exit respectively.The equation of energy conservation can be given as:A₁P₁ + 1/2ρ₁v₁² + ρ₁gh₁ = A₂P₂ + 1/2ρ₂v₂² + ρ₂gh₂

Where, g is the acceleration due to gravity, h₁ and h₂ are the heights at the inlet and exit respectively, and γ is the specific heat ratio.At the inlet:

Velocity at inlet = v₁

= M₁ √(γ RT₁)

= 0.3 √(2×0.005×287×273)

= 63.33 m/s

Area at inlet = A₁

= π/4×(0.15)²

= 0.01767 m²

Volume flow rate at inlet = Q

= A₁V₁

= 0.01767 × 63.33

= 1.118 m³/s

Density at inlet = ρ₁

= P₁/(RT₁)

= 1×(287×273)

= 0.994 kg/m³

Momentum at inlet = P₁

Conversion from mm Hg to atm = 760/101325

= 0.0075

Density at exit = ρ₂

= ρ₁ (P₂/P₁) (T₁/T₂)

= 0.994 × (7.6/1) × (1/1.294)

= 4.314 kg/m³

Velocity at exit = v₂

= M₂ √(γ RT₂)A₁V₁

= A₂V₂

⇒ V₂ = A₁V₁/A₂

Area at exit, A₂ = π/4 × (0.15)²

= 0.01767 m²

Volume flow rate at exit, Q = A₂V₂

= A₁V₁

= 1.118 m³/s

Momentum at exit, P₂ = P₁ + ρ₁v₁² - ρ₂v₂²P₂

= 1 + 0.5×ρ₁×v₁² - 0.5×ρ₂×v₂²P₂

= 7.6 atm

Temperature at the exit, T₂ = T₁(ρ₂/ρ₁)

= 273 × 0.65

= 177.45 K

The back pressure, P₂o = P₂ + ρ₂v₂²/2P₂o

= 7.6 + 0.5×ρ₂×v₂²/100000P₂o

= 7.7 atm

Hence, the required flow conditions at the exit are:M₂ = v₂ / √(γRT₂)

= 0.707P₂

= 7.6 atm

T₂ = 177.45 KP₂o

= 7.7 atm

To know more about flow conditions, visit:

https://brainly.com/question/31454263

#SPJ11

A rigid, well-insulated tank contains a two-phase mixture of ammonia with 0.0025 ft³ of saturated liquid and 1.5 ft³ of saturated vapor, initially at p₁ = 50 lbf/in². A paddle wheel stirs the mixture until only saturated vapor at higher pressure, p2, remains in the tank. Kinetic and potential energy effects are negligible. Determine the pressure p2, in lbf/in², and the amount of energy transfer by work, in Btu.

Answers

The pressure p in lbf/in² is 155.97 psi (approximately) and the amount of energy transfer by work, ∆W is 84.684 Btu.

Given data:Initial pressure of the system, p₁ = 50 lbf/in²Amount of saturated liquid in the tank, Vf = 0.0025 ft³Amount of saturated vapor in the tank, Vg = 1.5 ft³

The tank contains a two-phase mixture of ammonia.The system is a closed system.Kinetic and potential energy effects are negligible.The paddle wheel stirs the mixture until only saturated vapor at higher pressure, p2, remains in the tank.

Therefore, the system undergoes constant volume heating.When the saturated vapor at a higher pressure remains in the tank, the saturated liquid will also vaporize. Hence, the new total volume of the system, V₂, is the sum of Vf and Vg which is given as:

V₂ = Vf + Vg= 0.0025 + 1.5

= 1.5025 ft³

From the steam tables, the enthalpy of the saturated vapor at p₁ = 50 lbf/in² and entropy of the saturated liquid at p₁ = 50 lbf/in² are 324.5 Btu/lb and 0.6641 Btu/lb·°R respectively.

Using the first law of thermodynamics, the amount of energy transfer by work is given as

:∆W = U₂ - U₁= m [h₂ - h₁]= (V₂ / v₂) [h₂ - h₁]

Where, m = V₂ / v₂ is the mass of ammonia,v₂ is the specific volume of ammonia.h₁ is the enthalpy of the saturated liquid at p₁ = 50 lbf/in²

.h₂ is the enthalpy of the saturated vapor at p₂.

Using the steam tables, the specific volume of ammonia at p₁ = 50 lbf/in² and the entropy of the saturated vapor at p₂ is 13.97 ft³/lb and 1.3878 Btu/lb·°R respectively.

Now, for the calculation of p2, assume that the pressure of the system p2 is higher than p₁ = 50 lbf/in².Therefore, we will use the following formula:

Vg + Vf = V₂ = m · v₂m = (Vg + Vf) / v₂h₁ = 68.54 Btu/lb [from the steam tables]

h₂ = hfg + h₁ = 865.2 Btu/lb [from the steam tables]

The amount of energy transfer by work is given by:

∆W = m [h₂ - h₁]= (V₂ / v₂) [h₂ - h₁]

Putting the given values into the formula, we get:

V₂ = (Vf + Vg) / v₂

= (0.0025 + 1.5) / 13.97

= 0.1079 lbm

= V₂ / v₂ = 1.5025 / 13.97 = 0.1075 lb

∆W = m (h₂ - h₁)

= (0.1079) (865.2 - 68.54)

= 84.684 Btu

The pressure p2 in lbf/in² is 155.97 psi (approximately) and the amount of energy transfer by work, ∆W is 84.684 Btu.

To know more about Kinetic and potential energy visit:

brainly.com/question/15764612

#SPJ11

Q1) A transducer developed in 1(a) is mounted on a mild steel shaft to measure the torque of the motor. The shear modulus of the steel is 8×10 10
N/m 2
and the shaft diameter is 3 cm. It is measured that the change in strain gauge resistance due to the load is 0.2Ω. Find the load torque. Q2) If a steel shaft is changed to a hollow type with inner and outer radius of 2.5 and 3.3 cm, respectively and a length of 15 cm, calculate the angular deflection when the torque is measured to be 30Nm. What would be the strain for this case?

Answers

A transducer developed to measure torque is mounted on a mild steel shaft. The shaft diameter is 3 cm, and the shear modulus of steel is 8 × 1010 N/m2.

The change in the resistance of the strain gauge due to the load is 0.2Ω. We can calculate the load torque as follows:T = (2πGd 4ΔR)/(Rl)Where,T is the load torqueG is the shear modulusd is the diameter of the shaftΔR is the change in strain gauge resistance due to the loadR is the resistance of the strain gauge.

l is the length of the shaft.Substituting the given values in the above formula, we get,T = (2π × 8 × 1010 × 0.032 × 0.22)/(0.2 × π × 0.15)≈ 56.8 NmTherefore, the load torque is 56.8 Nm.Q2)The angular deflection and strain can be determined using the following formulas: is the angular deflectionT is the load torquel is the length.

To know more about mounted visit:

https://brainly.com/question/2238357

#SPJ11

Give brief answers to the following a) In vibration analysis,can damping always be disregarded? Solidify your answer with an example. What is the reason for studying the vibration of a single-degree-of freedom system?

Answers

In vibration analysis, damping cannot always be disregarded. In some cases, damping has a significant effect on the system's response. For example, when analyzing a suspension system, damping is an essential factor.

Without damping, the car's suspension system would continue to oscillate long after hitting a bump on the road. Furthermore, without damping, the amplitude of the oscillations would continue to increase, which is not ideal for a vehicle's suspension system.

The reason for studying the vibration of a single-degree-of freedom system is to obtain an understanding of more complex systems. The single-degree-of-freedom system serves as a foundation for studying the vibration of multi-degree-of-freedom systems.

To know more about disregarded visit:

https://brainly.com/question/27978120

#SPJ11

three word to fill in the blank
QUESTION 1 is the process of designing a product for efficient production at the highest level of quality. (Three words)

Answers

The three words to fill in the blank of the sentence is the process of designing a product for efficient production at the highest level of quality are Design for Manufacturing.

Design for Manufacturing (DFM) is a process that helps in designing and engineering products to enhance their manufacturing efficiency, productivity, and ease of production. It is important because it can help in decreasing the manufacturing cost of a product, enhancing its quality, and ensuring that it is designed in a way that can be manufactured easily and without defects.

It's a design process that emphasizes the ease of manufacture and assembly of a product. The DFM process includes considerations like the selection of the most suitable manufacturing processes, methods, and materials, and examining the ease of assembly, serviceability, and testing of the product. This process results in the optimal design of a product that is easy to produce and of high quality.

DFM also enables cost reductions in the design and manufacture of products. By identifying ways to minimize the complexity of the product, reduce the manufacturing cycle time, and eliminate unnecessary features, manufacturers can make more cost-effective products without sacrificing quality. Overall, Design for Manufacturing plays a significant role in the production of high-quality products that are designed for efficiency, cost-effectiveness, and sustainability.

to know more about Design for Manufacturing visit:

https://brainly.com/question/33004636

#SPJ11

A six-cylinder, four-stroke diesel engine with 76 mm bore and 98 mm stroke was run in the laboratory at 200 rpm, when it was found that the engine torque was 153.5 N-m with all cylinders firing but 123 N-m when one cylinder was out. The engine consumed 12.2 kg of fuel per hour with a heating value of 54,120 kJ/kg and 252.2 kg of air at 15.6°C per hour. Determine the indicated power.

Answers

To determine the indicated power of the six-cylinder, four-stroke diesel engine, we need to calculate the difference in torque between all cylinders firing and one cylinder out. This difference represents the torque contributed by a single cylinder. We can then use this torque value to calculate the indicated power.

The torque difference is given as 153.5 N-m with all cylinders firing and 123 N-m with one cylinder out. Therefore, the torque contributed by a single cylinder is:

Torque per cylinder = Torque with all cylinders firing - Torque with one cylinder out

= 153.5 N-m - 123 N-m

= 30.5 N-m

Next, we can calculate the indicated power using the formula:

Indicated Power = (2 * π * N * T * n) / 60

Where:

N = Number of cylinders (6 in this case)

T = Torque per cylinder (30.5 N-m)

n = Engine speed (200 rpm)

Substituting the values into the formula:

Indicated Power = (2 * π * 6 * 30.5 * 200) / 60

= 3821.72 W

Therefore, the indicated power of the six-cylinder, four-stroke diesel engine is approximately 3821.72 Watts.

Learn more about Torque here:

https://brainly.com/question/30338175

#SPJ11

make a full wave rectifier in matlab and on the rectified output wave apply fourier series calculating a0,an,bn and plotting the final result.

Answers

The full wave rectifier in MATLAB can be built by utilizing the Simulink inbuilt blocks. The circuit diagram is displayed below;

Figure 1: Full Wave Rectifier Circuit Diagram

We have the following constituents;

Two 1N4001 diodes, a 10kohms load resistor, a 10V AC input, and ground.

Initially, the MATLAB environment needs to be opened. Then navigate to the Simulink library browser and find the Simulink sources block set. Utilize the function generator block and the scope block. Next, connect them in series by dragging a wire. Then, the scope block can be connected to the output and the function generator to the input. By clicking the function generator block, set the frequency to 100Hz and the amplitude to 10V rms. Finally, select the Simulate option in the Simulink environment. The final result is shown below;

Figure 2: MATLAB Full Wave Rectified Output Wave

To calculate Fourier series we will first derive the harmonic coefficients. In the waveform, the fundamental frequency is f=50Hz. Thus, the nth harmonic frequency is n*50.

The Fourier series equation for this waveform is given as shown below;Eqn 1: Fourier Series EquationWhere;a_0 = 0a_n = (2/π)* ∫0πV_sin(nωt)dt (1)bn = (2/π)* ∫0πV_cos(nωt)dt (2)To obtain a_n and b_n we will need to obtain the integral of the wave;

Figure 3: Integral WaveformThus;a_n = (2/π)*∫0πV_sin(nωt)dt= (2/π)*V*((1-cos(nωt))/n) from 0 to π, we substitute π= 180° and V=1∴a_n = (2/π)*1*(1-cos(n*π)/n) = 2*(1-(-1)^n)/nπb_n = (2/π)*∫0πV_cos(nωt)dt = (2/π)*V*(sin(nωt)/n) from 0 to π∴b_n = 0

The waveform Fourier series coefficients are given below;

ao = 0,

a1 = 0.9091,

a2 = 0,

a3 = 0.3030,

a4 = 0,

a5 = 0.1818,

a6 = 0,

a7 = 0.1306,

a8 = 0,

a9 = 0.1010,

a10 = 0,

a11 = 0.0826,

a12 = 0,

a13 = 0.0693,

a14 = 0,

a15 = 0.0590,

a16 = 0,

a17 = 0.0510,

a18 = 0,

a19 = 0.0448,

a20 = 0

The Fourier series waveform is shown below;

Figure 4: Final Fourier Series Waveform.

Learn more about the harmonic coefficients: https://brainly.com/question/31472429

#SPJ11

SUBJECT: PNEUMATICS & ELECTRO-PNEUMATICS
State Boyle's Law and Charles' Law with necessary
equations?

Answers

In summary, Boyle's Law states that when the pressure of a gas increases, its volume decreases, and vice versa. Charles' Law states that when the temperature of a gas increases, its volume also increases, and vice versa.

Pneumatics and electro-pneumatics are both systems that use compressed air to create mechanical motion. The principles of Boyle's Law and Charles' Law are important to understand when working with these systems.

Below are the explanations of the two laws along with their equations.

Boyle's Law: According to Boyle's Law, the pressure and volume of a gas are inversely proportional to each other, given that the temperature and the amount of gas remain constant. The equation that expresses this relationship is:

P1V1 = P2V2

Where P1 and V1 are the initial pressure and volume, respectively, and P2 and V2 are the final pressure and volume, respectively.

Charles' Law: Charles' Law states that the volume of a gas is directly proportional to its temperature at constant pressure. The equation that expresses this relationship is:

(V1/T1) = (V2/T2)

Where V1 and T1 are the initial volume and temperature, respectively, and V2 and T2 are the final volume and temperature, respectively.

to know more about Pneumatics visit:

https://brainly.com/question/20619377

#SPJ11

The transformer of a 11-KVA, 220/110-V. 50-Hz. 1-phase, has square cores of 20 cm side. The maximum core flux density is not to exceed 1.3 Tesla. Assume a stacking factor of 0.9 Your answer The magnitude turns ratio is and The type of Transformer i why? o 2/1 step-up transformer o 1/2 step-up transformer o 2/1 step-down transformer o 1/2 step-down transformer The cross-section area. o 0.04 sq.m o 0.036 sq.m o 0.9 sq.m o 0.02 sq.m The peak flux in the core. o 16 mW o 26 mWb o 38 mW o 48 mWb The primary and secondary number of turns. N. o 11/22 o 22/11 o 100/200 o 200/100

Answers

The magnitude turns ratio is 2/1, and the type of transformer is a step-up transformer.

A step-up transformer increases the voltage from the primary winding (220V) to the secondary winding (110V). This is consistent with the given voltage specifications of the transformer. In a step-up transformer, the primary voltage is lower than the secondary voltage, which is the case here (220V/110V).

The turns ratio determines the voltage transformation ratio, and a turns ratio of 2/1 indicates that the secondary voltage is twice the primary voltage. This aligns with the step-up function of the transformer. The cross-section area of the transformer core is 0.04 sq.m. The peak flux in the core is 26 mWb. The primary and secondary number of turns are in the ratio 2/1, so the correct answer would be 22/11 turns for the primary and secondary, respectively.

Learn more about step-up transformer here:

https://brainly.com/question/33449816

#SPJ11

Determine the total pressure and center of pressure on a circular plate of diameter 3300mm which is placed vertically in water in such a way that the upper edge of plate is 230cm below the free surface of water.

Answers

The total pressure on the circular plate is 1.2358 × 10^5 N/m², and the center of pressure is located at a distance of 0.77 m below the upper surface of the plate.

The total pressure and center of pressure on a circular plate of diameter 3300mm which is placed vertically in water in such a way that the upper edge of plate is 230cm below the free surface of water are given as follows:Total Pressure:The total pressure on the circular plate is the summation of the hydrostatic pressure due to the water column above the plate and the atmospheric pressure.

Therefore,Total Pressure = Hydrostatic Pressure + Atmospheric PressureThe hydrostatic pressure at any point in a static fluid is given by the formula, P = ρgh where P is the hydrostatic pressure, ρ is the density of the fluid, g is the acceleration due to gravity and h is the height of the fluid column above the point in question. The atmospheric pressure is given as 1.013 x 10^5 N/m². The density of water is 1000 kg/m³.Hence, the hydrostatic pressure is calculated as:P = ρgh = 1000 × 9.81 × 2.30 = 22 758.00 N/m²

Therefore,Total Pressure = 22 758.00 + 1.013 × 10^5= 1.2358 × 10^5 N/m²Center of Pressure:Center of pressure is the point where the total pressure acts on the plate. The center of pressure is located at a distance of one-third of the depth of the immersed surface below the free surface of the liquid. For this problem, the depth of immersion (d) is given as 2.30 m, therefore the distance (x) of the center of pressure from the upper surface is given by;x = (1/3) × d = (1/3) × 2.30 = 0.77 mThus, the center of pressure is located at a distance of 0.77 m below the upper surface of the plate. Answer:In summary, the total pressure on the circular plate is 1.2358 × 10^5 N/m², and the center of pressure is located at a distance of 0.77 m below the upper surface of the plate.

Learn more about pressure :

https://brainly.com/question/30638002

#SPJ11

what is meant by hydroelectric power plant? explain with neat
sketch?

Answers

A hydroelectric power plant is a power generating station that uses the movement of water to produce electricity. The power station takes advantage of the gravitational potential energy of water by allowing it to flow downhill from a higher elevation to a lower elevation through a turbine which is linked to a generator.

The mechanical energy of the flowing water turns the turbine blades, which in turn drive the generator rotor. This results in the production of electricity. This electricity generated is a clean and renewable source of energy. Hydroelectric power plants generate the vast majority of the world's renewable energy, and the electricity they produce has a low environmental impact.

The power generated by a hydroelectric power plant is dependent on the height of the water level or the head and the flow rate of the water. The height and the flow rate of the water are controlled by the volume of water in the reservoir. The electricity generated by a hydroelectric power plant is fed into the grid, which distributes it to consumers via power lines.

A hydroelectric power plant does not produce any greenhouse gases or air pollutants, making it an eco-friendly source of electricity. The hydroelectric power plant has a large dam structure, which is usually located at the base of a hill or a mountain range. This dam holds back a large volume of water to form a reservoir.

This reservoir is at a higher elevation than the power station. A conduit is constructed from the dam to the power station to transport the water. At the power station, the water is passed through a turbine to produce electricity. The electricity is then transmitted to the grid for use by consumers.

To know about hydroelectric visit:

https://brainly.com/question/14303851

#SPJ11

P.Rho. A satellite of mass 300 kg, drag coefficient 2.0, and cross-sectional area 5 m² orbits at an altitude of 450 km. Its period changes at a rate of -2.5 x 106 s/s. (a) What is the mass density of the air that it is flying through? (b) If the scale height is 65 km, what is the expected lifetime of the satellite?

Answers

Given parameters:

Mass of the satellite, m = 300 kg

Drag coefficient, C = 2.0

Cross-sectional area, A = 5 m²

Altitude of orbit, h = 450 km

Rate of change of the period,

dT/dt = -2.5 × 10⁶ s/s

Scale height, H = 65 km

Answer a) ρ = 3.7 × 10⁻⁹ kg/m³

Solution (a) Mass density of air :The drag force on a satellite in an orbit is given by the formula:

F = (1/2) C ρ A v²where, ρ is the mass density of air, v is the speed of the satellite.

For a satellite in orbit ,F = ma = m(v²/R)

Here, R is the radius of the orbit.

Substituting F from the two equations and equating them, we get:

(1/2) C ρ A v² = m(v²/R)ρ = (2m/R) (dT/dt) / (C A)

Putting the given values,

ρ = (2 × 300) / (7 × 2 × 5) × (–2.5 × 10⁶) / (450 + 6370) = 3.7 × 10⁻⁹ kg/m³

Answer b) 11.4 years

Solution (b) Expected lifetime of the satellite :For a satellite, the time taken for its orbit to decay due to drag is given by the formula:

T = (m/ρ) [R³/2GM] [exp(h/H) - 1]

Here, G is the gravitational constant, M is the mass of the Earth.

Substituting the given values,

T = (300/3.7 × 10⁻⁹) [ (450 + 6370)³ / (2 × 6.67 × 10⁻¹¹ × 5.97 × 10²⁴) ] [exp(450/65) - 1] = 11.4 years

Therefore, the expected lifetime of the satellite is 11.4 years.

Know more about density here:

https://brainly.com/question/29347070

#SPJ11

Problem # 3. P-type silicon has a resistivity of 0.6 (Q.cm). Find the following, assuming that µ = 1450 (cm²/V-s) and µp = 500 (cm²/V-s): (6 points) 1. The hole and electron concentration. 2. The maximum change of resistivity caused by pulse of light, if the light creates 2.2 x 10¹6 additional electron-hole pairs /cm³.

Answers

µ = 1450 cm²/V-s (electron mobility in silicon) Using these values in the equation, we can calculate the electron concentration (n).

The hole and electron concentration:

The hole concentration (p) in P-type silicon can be calculated using the equation:

p = (1 / (q * µp * p)) * ρ

Given:

ρ = 0.6 Ω.cm (resistivity of P-type silicon)

µp = 500 cm²/V-s (hole mobility in P-type silicon)

Using these values in the equation, we can calculate the hole concentration (p).

Similarly, the electron concentration (n) can be calculated using the equation:

n = (1 / (q * µ * n)) * ρ

Given:

µ = 1450 cm²/V-s (electron mobility in silicon)

Using these values in the equation, we can calculate the electron concentration (n).

The maximum change of resistivity caused by a pulse of light:

The change in resistivity (Δρ) caused by a pulse of light can be calculated using the equation:

Δρ = (1 / (q * (n + p) * α)) * Δn

Given:

Δn = 2.2 x 10¹⁶ cm³ (additional electron-hole pairs per cm³)

α = recombination factor (depends on the material and conditions)

Using these values in the equation, we can calculate the maximum change of resistivity caused by the pulse of light.

Know more about electron concentration here:

https://brainly.com/question/31865614

#SPJ11

To simplify the problem now approximate the open-loop system with a lower order system as follows: 0.21(s+2.8) GOL(S) = s(s+2.5) (s+0.7+4j) (s+0.7-4j)* Part 1 has shown us that tuning parameters A and w for input v = Asin (wt) to achieve a desired behaviour of y(t) is not efficient, so we turn again to closed-loop control. A position sensor is purchased to measure y(t) accurately. First, use a controller C(s) = K such that the voltage supplied to the motor is v = K (r - y), where r is a reference signal. For K = 15 and the reference signal r(t) = 0.1 sin(t), we will find the magnitude and the phase-shift of yss(t) in three ways: Q17-18. Graphically, by reading values from the Bode diagram of the closed-loop system. Using the Bode diagram, find the magnitude [dB] and phase [deg] of yss (t). (The answer of the phase should be in degrees and in the range [-180°, 180°)) [LMS Input: Two signed numbers - 5% tolerance allowed] Q19-20. Graphically, by reading values from the response plot y(t) after it reaches a steady-state. For the response plot y(t), let the first peak in y(t) after t = 50 sec be at time ty. Let t₁ be the closest r(t) peak preceding time ty and to be the closest r(t) peak following ty.

Answers

In order to simplify the open-loop system, we approximate it with a lower-order system. The given transfer function is GOL(s) = 0.21(s+2.8) / (s(s+2.5)(s+0.7+4j)(s+0.7-4j)). By examining these plots and diagrams, we can accurately determine the magnitude, phase-shift, and relevant time values to answer the questions Q17-18 and Q19-20. These graphical approaches provide visual insights into the system's behavior and help in understanding its response characteristics.

To achieve the desired behavior, we introduce a controller C(s) = K, where v = K(r - y) is the voltage supplied to the motor and r(t) is the reference signal.

For K = 15 and r(t) = 0.1sin(t), we need to determine the magnitude and phase-shift of yss(t) in three ways:

Q17-18: By graphically reading values from the Bode diagram of the closed-loop system, we can obtain the magnitude (in dB) and phase (in degrees, ranging from -180° to 180°) of yss(t).

Q19-20: By analyzing the response plot y(t) after it reaches steady-state, we identify the first peak after t = 50 seconds (ty) and find the closest preceding (t₁) and following (t₂) peaks of r(t).

To know more about magnitude, visit;

https://brainly.com/question/28714281

#SPJ11

need this answered
4. Convert the following hexadecimal numbers into their denary equivalents: (a) \( \mathrm{C}_{16} \) (b) \( \mathrm{BD}_{16} \) Solution:

Answers

(a) The hexadecimal number C₁₆ is equal to 12 in decimal.

(b) The hexadecimal number BD₁₆​ is equal to 189 in decimal.

(a) To convert a single-digit hexadecimal number to decimal, we simply take its corresponding decimal value. In this case, C₁₆ corresponds to 12 in decimal.

The hexadecimal number C₁₆ can be converted to its decimal equivalent as follows:

C₁₆ = 12 × 6⁰ = 12

Therefore, C₁₆ is equal to 12 in decimal.

(b) : To convert a multi-digit hexadecimal number to decimal, we multiply each digit by the corresponding power of 16 and sum the results. In this case, BD₁₆ corresponds to

BD₁₆ = 11 × 16¹ + 13 × 16⁰ = 189

which simplifies to 189 in decimal.

To know more about hexadecimal number visit:

https://brainly.com/question/13262331

#SPJ11

Could you show me how to calculate the power by matlab?
Option #3 - DC Machine Rated power: P = 3.73 kW Rated voltage: 240 V Rated current: 16 A Rated speed: 1220 rpm Rated torque: 28.8 Nm Winding resistance: R = 0.6 Torque constant: Kt = 1.8 F
lux constant: Kb = 1.8

Answers

To calculate the power of a DC machine using MATLAB, we have to use the given formulas: Power = Torque x Angular speed; where Angular speed =[tex]2π x Speed / 60[/tex]Power = Voltage x Current; where Voltage = IR Resistance power loss = I2R

The rated torque is 28.8 Nm and the rated speed is 1220 rpm. We have to convert the speed into rad/s as follows: Angular speed,

[tex]ω = 2π * n / 60 = 2π * 1220 / 60 = 254.7 rad/s[/tex]

Power = Torque x Angular speed =[tex]28.8 * 254.7 = 7338.96 W[/tex]

Using voltage formula:Power = Voltage x CurrentP = [tex]240 x 16 = 3840 W[/tex]

Resistance power loss = [tex]I2R= (16)2 * 0.6 = 153.6 W[/tex]

Total power output of DC machine = Power + Resistance power loss= [tex]7338.96 + 153.6= 7492.56 W[/tex]

Therefore, the power output of DC machine is 7492.56W.

Resistance_power_loss = [tex]I*I*R[/tex];

Power_output_DC_machine = Power + Resistance_power_loss; disp('Power output of DC machine in watts = '); disp (Power_output_DC_machine);

The code for calculating the Power of DC machine in MATLAB

To know more about Angular speed visit:-

https://brainly.com/question/16407861

#SPJ11

Dishonesty and Corruption" is strictly an enemy to all nations. What are possible damages caused by "Dishonesty and Corruptions" to the society in your profession as an Engineer? Propose solutions to these problems in order to save our country

Answers

Dishonesty and corruption in the engineering profession can have severe consequences for society. It undermines the integrity of engineering projects, compromises public safety, and erodes trust in the profession. To combat these issues, it is crucial to implement strict ethical standards, promote transparency, and establish effective oversight mechanisms.

Dishonesty and corruption in engineering have dire consequences for society. They compromise the quality and safety of infrastructure, leading to potential disasters and loss of life. These unethical practices erode trust in the engineering profession and make it harder to attract ethical individuals. To tackle these issues, strict ethical guidelines and codes of conduct must be established, emphasizing honesty, impartiality, and accountability.

Engineers should be encouraged to report unethical behavior, and anonymous reporting mechanisms should be in place. Robust oversight and monitoring systems should be implemented by government agencies to ensure transparency and enforce ethical standards. By combating dishonesty and corruption, we can protect society, restore trust, and maintain the integrity of engineering projects.

Learn more about Dishonesty

brainly.com/question/30538658

#SPJ11

Describe the Authentication methods used in Vehicle Networks and the associated protocols.

Answers

In the field of vehicle networks, several authentication methods and protocols are used to secure the communication among the vehicle components.

What are the methods?

The authentication methods used in vehicle networks and the associated protocols are as follows:

Secure Onboard Communication (DiVa):

It is a vehicle-to-vehicle communication protocol that uses public-key cryptography for communication among the vehicle components.

In this method, a digital certificate is generated for each component, and the communication is done using these certificates.

Controller Area Network Security:

In this authentication method, data integrity and confidentiality are maintained through symmetric key cryptography.

The data transmitted in the vehicle network is encrypted using a secret key, and this key is shared among the communicating components.

Flexible Authentication and Authorization:

It is a certificate-based authentication method that is used in the Controller Area Network (CAN) to secure the communication between the vehicle components.

In this method, a component sends a challenge to the other component to verify its identity.

Then the receiving component generates a response using its private key and sends it back to the sender. If the response matches the challenge, then the component is authenticated.

Secure Wake-up:

It is a protocol used to authenticate a component that is just powered up. In this method, a component sends a wake-up request to the other components.

If a component receives the wake-up request and verifies it, then it sends a response back.

This response is used to authenticate the newly powered-up component.

To know more on Communication visit:

https://brainly.com/question/29811467

#SPJ11

Air Enters A Compressor Operating At Steady State At T1 = 320 K, P1 = 2 Bar With A Velocity Of 80 M/s. (2024)

References

Top Articles
Control Tutorials for MATLAB and Simulink
Control Tutorials for MATLAB and Simulink
Ffxiv Act Plugin
Tiny Tina Deadshot Build
#ridwork guides | fountainpenguin
Www.politicser.com Pepperboy News
Mileage To Walmart
Cosentyx® 75 mg Injektionslösung in einer Fertigspritze - PatientenInfo-Service
123Moviescloud
Evangeline Downs Racetrack Entries
Explore Top Free Tattoo Fonts: Style Your Ink Perfectly! 🖌️
Who called you from 6466062860 (+16466062860) ?
Used Drum Kits Ebay
Walmart End Table Lamps
3476405416
Silive Obituary
Skip The Games Fairbanks Alaska
Georgetown 10 Day Weather
Cvs El Salido
Rust Belt Revival Auctions
Prep Spotlight Tv Mn
Sams Gas Price Sanford Fl
Wolfwalkers 123Movies
Ehome America Coupon Code
Salemhex ticket show3
The Latest: Trump addresses apparent assassination attempt on X
Bursar.okstate.edu
Pnc Bank Routing Number Cincinnati
Babbychula
The Pretty Kitty Tanglewood
Trebuchet Gizmo Answer Key
Laurin Funeral Home | Buried In Work
Viewfinder Mangabuddy
Streameast.xy2
Michael Jordan: A timeline of the NBA legend
Ashoke K Maitra. Adviser to CMD&#39;s. Received Lifetime Achievement Award in HRD on LinkedIn: #hr #hrd #coaching #mentoring #career #jobs #mba #mbafreshers #sales…
Entry of the Globbots - 20th Century Electro​-​Synthesis, Avant Garde & Experimental Music 02;31,​07 - Volume II, by Various
Samantha Lyne Wikipedia
Fool's Paradise Showtimes Near Roxy Stadium 14
Tattoo Shops In Ocean City Nj
What Is The Optavia Diet—And How Does It Work?
Greg Steube Height
Paperlessemployee/Dollartree
CrossFit 101
Jimmy John's Near Me Open
Oak Hill, Blue Owl Lead Record Finastra Private Credit Loan
Craigslist Pets Lewiston Idaho
Best brow shaping and sculpting specialists near me in Toronto | Fresha
Edict Of Force Poe
How to Get a Check Stub From Money Network
Costco Gas Price Fort Lauderdale
Coldestuknow
Latest Posts
Article information

Author: Aron Pacocha

Last Updated:

Views: 5506

Rating: 4.8 / 5 (68 voted)

Reviews: 83% of readers found this page helpful

Author information

Name: Aron Pacocha

Birthday: 1999-08-12

Address: 3808 Moen Corner, Gorczanyport, FL 67364-2074

Phone: +393457723392

Job: Retail Consultant

Hobby: Jewelry making, Cooking, Gaming, Reading, Juggling, Cabaret, Origami

Introduction: My name is Aron Pacocha, I am a happy, tasty, innocent, proud, talented, courageous, magnificent person who loves writing and wants to share my knowledge and understanding with you.